Questions tagged [decidability]

The tag has no usage guidance.

46 questions with no upvoted or accepted answers
Filter by
Sorted by
Tagged with
17 votes
0 answers
779 views

Decidability of $x^3+y^3+z^3 = c$

I wondering if it is known whether the following problem is algorithmically decidable or undecidable by Turing machines: given an integer c, determine if there are integers $(x,y,z)$ such that $x^3+y^...
Anonymous's user avatar
  • 171
16 votes
0 answers
327 views

Is tightness decidable?

Given a contact structure on a three-manifold, is there an algorithm to decide whether or not it tight? For concreteness' sake, let's agree to represent the given contact three-manifold via an open ...
John Pardon's user avatar
  • 18.3k
8 votes
0 answers
232 views

Hilbert 10th problem for genus 2 equations

Hilbert 10th problem, while undecidable in general, remains open for 2-variable equations: we do not know if there is an algorithm that, for polynomial $P(x,y)$ with integer coefficients, decides ...
Bogdan Grechuk's user avatar
8 votes
0 answers
111 views

The conjugacy problem for two-relator groups

Is the conjugacy problem for two-relator groups known to be undecidable? The word problem for two-relator groups is a famous open problem (appearing e.g. as Question 9.29 in the Kourovka notebook), ...
Carl-Fredrik Nyberg Brodda's user avatar
8 votes
0 answers
252 views

Membership problem in general linear group

This is surely a very well known problem, but I could not find an answer on MO or on Google, so here I am. Given some finitely generated free subgroup $H$ of $\operatorname{GL}_n(\mathbb{Z}[t,t^{-1}])...
user8253417's user avatar
7 votes
0 answers
261 views

Uniform word problem in finitely presented simple groups

The following question arose in the comments on this question, and it seems like a reasonable question to ask in its own right. I've added some additional details. The word problem in any fixed ...
Carl-Fredrik Nyberg Brodda's user avatar
7 votes
0 answers
271 views

Is decidability reducible to unique decidability (perhaps in multilinear polynomial situations)?

Given a Diophantine equation it is not decidable if it has integer solution. I. Is there a Diophantine set $\mathcal D_{unique}$ satisfying the properties every member in $\mathcal D_{unique}$ is a ...
Turbo's user avatar
  • 13.7k
7 votes
0 answers
607 views

Proving Richardson's theorem for constants

(I asked this a little over 3 months ago on math.SE, and when I initially re-asked here, no one had responded there. $\:$ After I re-asked here, Eric Towers responded there, since I had forgotten to ...
user avatar
5 votes
0 answers
203 views

Integer points of rational function in 2 variables

Is there an algorithm that, given polynomials $P(x)$ and $Q(y)$ with integer coefficients, decides whether there exists integers $x$ and $y$ such that $\frac{P(x)}{Q(y)}$ is an integer? This is a ...
Bogdan Grechuk's user avatar
5 votes
0 answers
162 views

Example of applying real quantifier elimination algorithm for polynomials

Sorry if any of this is unclear, or doesn't make much sense, I'm still trying to figure it out, a practical example such as this would likely help me understand better than anything else. I have read ...
Evan's user avatar
  • 51
5 votes
0 answers
282 views

Is the two variable fragment of arithmetic, i.e., theory of ($\mathbb{N}, + ,\times$), decidable?

Any references would be appreciated. Most places only address different vocabularies (e.g. a survey of arithmetical definability by Bes).
Thinniyam Srinivasan Ramanatha's user avatar
4 votes
0 answers
157 views

Undecidability for hyperbolic Wang-tilings - pentagons, heptagons, octagons, oh my!

Berger proved that the problem of determining if a finite set of Wang tiles can tile the plane is undecidable. Robinson reproved Berger's result and raised the question of considering the ...
user101010's user avatar
  • 5,319
4 votes
0 answers
155 views

Subgroup membership problem for Noetherian groups

I am interested in the status of the subgroup membership problem (MP) for finitely presented Noetherian groups. That is, given a finite presentation $\langle X,R\rangle$ for a Noetherian group, \begin{...
suitangi's user avatar
  • 333
4 votes
0 answers
347 views

minimum size of undecidable quadratic diophantine problems

According to Matiyasevich, the existence of integer solutions of systems of polynomial equations with integer coefficients is undecidable. By introducing additional variables substituting factors of ...
Arnold Neumaier's user avatar
4 votes
0 answers
112 views

Deciding equality in free models of a (generalized) Lawvere theory

Let $F : \mathcal{C} \rightarrow \mathcal{D}$ be functor of Lawvere theories $\mathcal{C}, \mathcal{D}$ (i.e. cartesian categories where every object is isomorphic to some power of a chosen object) ...
Martin Bidlingmaier's user avatar
4 votes
0 answers
154 views

Undecidability of the existential theory

Do you know if I can find the proof that the existential theory of $\mathbb{Z}$ with the structure of addition , divisibility and the relation $(\exists s \in \mathbb{Z})m=np^s$ is undecidable, ...
Mary Star's user avatar
  • 299
4 votes
0 answers
212 views

The theory of two finite linear orders

My colleague Matthias Baaz is looking for a reference for the following question (or possibly theorem): Let T be the "theory of pairs of finite linear orders". That is, consider all finite ...
Goldstern's user avatar
  • 13.9k
3 votes
0 answers
49 views

Maximal number of aperiodic Wang tiles

I was wondering whether there is an analogue result to the minimality of Wang tiling, in the direction of maximality. I think that the paper by Jeandel and Rao, shows that the minimal number of Wang ...
Keen-ameteur's user avatar
3 votes
0 answers
96 views

Decidability of theory of modules over a ring of finite representation type

I have read from Mike Prest's model theory for modules (London lecture note series) chapter 17 that a Ring of finite representation type has a decidable theory of modules. Here decidability was ...
Yoneda Lemma's user avatar
3 votes
0 answers
148 views

Why is the proof of decidability of arithmetic (Theorem 2.1) in Hamkins & Lewis (2000) enough?

Recently, I was reading the paper "Infinite Time Turing Machines" by Hamkins & Lewis. And one of the first theorems (Theorem 2.1) is about decidability of arithmetic. The proof is quite ...
Jeremy's user avatar
  • 31
3 votes
0 answers
114 views

Variation in decidability of diophantine equations with field extension

Let $O_k$ be the ring of integers in a subfield $k$ of $\overline{\mathbb{Q}}$. Let's call an equation $p(x_1, \dots, x_n) = 0$ where $p$ is a polynomial in $n$-variables $x_1, \dots, x_n$ with ...
Fanta's user avatar
  • 31
3 votes
0 answers
116 views

Post correspondence problem: Busy beaver variant

The Post correspondence problem (Wikipedia link) is to decide for $k$ pairs of strings $$(a_1,b_1), (a_2, b_2), ..., (a_k,b_k),$$ if there exists a finite sequence of numbers $c_j, 0\le j\le j_\max $ ...
Thomas's user avatar
  • 2,691
3 votes
0 answers
193 views

Can we "invert" Diophantine equations such as $x^3+y^3+z^3=k$ in to halting probabilities for some universal Turing machine?

Following Poonen [1], Davis[2], Chaitin [3], and Ord and Kieu [4]: Is it possible that there is a polynomial $P$ of degree $d\le 4$, along with a prefix-free universal Turing machine $T$, such that ...
Mark S's user avatar
  • 2,143
3 votes
0 answers
57 views

Decidability of first order theory of subclasses of posets

Is the first order theory of finite posets known to be undecidable? Does anyone know a survey about such results?
Thinniyam Srinivasan Ramanatha's user avatar
3 votes
0 answers
174 views

What is the general feeling for Hilbert's 10th problem for Q?

We know that Hilbert's 10th problem for $\mathbb{Z}$ is undecidable. I was wondering whether there is a strong opinion in the mathematical community on the decidability of Hilbert's 10th for $\mathbb{...
user47386's user avatar
2 votes
0 answers
70 views

Is parquetability decidable?

Let $T\neq \emptyset$ be a finite subset of $\mathbb{Z}\times\mathbb{Z}$. We say that $\mathbb{Z}^2 = \mathbb{Z}\times\mathbb{Z}$ is parquettable by $T$ if there is a partition $\frak P$ of $\mathbb{Z}...
Dominic van der Zypen's user avatar
2 votes
0 answers
63 views

Decidability of the solvability of quadratic systems

Let a finite collection of (complex) unknowns $\{x_1,\ldots,x_n\}$ be given, as well as an affine system $AX=B$ in the quadratic variables $X:=[x_i x_j : i\leq j]$, with entries in a computable ...
Loïc Teyssier's user avatar
2 votes
0 answers
137 views

Compare my software's representation of exponential numbers and 0?

Suppose I have a real number $$ x=\sum_{i=1}^n a_i e^{\lambda_i} $$ where $a_i,\lambda_i$s are complex algebraic numbers. Is there an algorithm to determine whether it is greater than 0 or less than ...
gondolf's user avatar
  • 1,493
2 votes
0 answers
213 views

The elementary theory of finite commutative rings

I have wondered the decidability of elementary theory of finite commutative rings. Since we know that the elementary theory of finite fields is decidable shown by J.Ax (The Elementary Theory of Finite ...
Max CYLin's user avatar
  • 151
2 votes
0 answers
89 views

Is the equational theory of the variety of ternary self-distributive algebras decidable?

A ternary self-distributive algebra is an algebra $(X,t)$ that satisfies the identity $$t(u,v,t(x,y,z))=t(t(u,v,x),t(u,v,y),t(u,v,z)).$$ Is the equational theory of the variety of ternary self-...
Joseph Van Name's user avatar
2 votes
0 answers
89 views

Matrix (geometric sum) orbit problem

Is the following algorithmic problem known to be decidable/undecidable? Input: an element $\mathbf{v} \in \mathbb{Z}^n$, a matrix $\mathbf{A} \in GL_n(\mathbb{Z})$, and a subgroup $H \leqslant \...
suitangi's user avatar
  • 333
2 votes
0 answers
111 views

Positive existential theory of $(\mathbb{Z}; +, |_n)$

I am reading a paper and there is the following theorem: Let $n$ be a fixed integer, and $n >1$. Denote divisibility in $\mathbb{Z}[\frac{1}{n}]$ by $|_n$, thus for all $x, y \in \...
Mary Star's user avatar
  • 299
2 votes
0 answers
46 views

Relation between indexed languages (OI-macro or context-free tree) and scattered context languages

I'm not sure about the relation between indexed languages (generated by indexed grammars--Aho) and scattered context languages (generated by scattered context grammars--J Hopcroft). I think that ...
Nate's user avatar
  • 21
2 votes
0 answers
135 views

Extend Lowenheim's decidability result to fragment of second-order logic

Since relational monadic first-order logic has finite model property, its SAT problem is decidable. In H.Behmann's paper, he extended this result to fragment of SOL where all predicates, free and ...
Infinity's user avatar
1 vote
0 answers
66 views

Decidability of a polynomial-exponential equation in two variables

My question is with regards to the following (algorithmic) problem: Problem. Given $f\in \mathbb{Z}[x,y], a,b\in \mathbb{Q}, r\in \mathbb{Z}$, do there exist positive integers $m,n$ such that $f(m,n) =...
thebogatron's user avatar
1 vote
0 answers
101 views

Game with Turing machines

Introduction The following game is quite nice: Alice has, in secret, constructed a polynomial $P \in \mathbb{Z}[x]$. On day $n=1,2,3,...$, she secretly writes down $P(n)$ on a piece of paper. Each day,...
Per Alexandersson's user avatar
1 vote
0 answers
72 views

Non-degenerate solutions in multiplicative subgroups of $\mathbb{Q}$ of finite ranks

I am trying to study whether of a set of first order sentences is decidable, and the key is to figure out an algorithm to compute all non-degenerated solutions. The setting is as follow. Let $q = (q_{...
user978394's user avatar
1 vote
0 answers
76 views

Decidability of linear equation about Sine and Cosine

Given integers $n,d$, and rational numbers $a_i,b_i,l_{i,j},s_{i,j}$ for $1\leq i\leq d$, $1\leq j\leq n$, we are considering the following equation $$ \sum_{i} [a_i \sin(\sum l_{i,j}\theta_j)+b_i \...
gondolf's user avatar
  • 1,493
1 vote
0 answers
55 views

Equality of combinations of exponentials and logarithms

Suppose I have some combination of exponentials, logarithms, and arithmetic operations on rational numbers. For example, $e^{e^{r_1} + \log r_2} - r_3$. Under what conditions does an algorithm exist ...
pavpanchekha's user avatar
  • 1,461
1 vote
0 answers
90 views

determine the existence of positive semi-definite matrix

Given a $d\times d$ complex matrix subspace $S$, we are asking whether there is some finite integer $n$ such that there exists a non-zero positive semi-definite matrix is orthogonal to $S^{\otimes n}$....
gondolf's user avatar
  • 1,493
1 vote
0 answers
114 views

What is the generic complexity of First Order Predicate Calculus?

I suspect that it should be the same as that of the Turing machine halting problem, which wikipedia gives as GenP and attributes this result to Hamkins and Miasnikov, but I am not sure. Is the generic ...
Jim Graber's user avatar
1 vote
0 answers
87 views

unique types and decidability

Suppose $\mathcal{M}$ is an infinite structure which has the property that every type that is realised is realised uniquely. Also assume that every element of $\mathcal{M}$ is definable but there is ...
Thinniyam Srinivasan Ramanatha's user avatar
0 votes
0 answers
117 views

Genus $0$ algebraic curves integral points decidable?

It is known there is an explicit algebraic variety in $\mathbb Z[x_1,\dots,x_t]$ a bounded $t>2$ whose integral zero-set is non-empty is undecidable. If the variety has genus $0$ is there anything ...
Turbo's user avatar
  • 13.7k
0 votes
0 answers
96 views

Multivariate polynomial with infinite but discrete roots on one variable

I want to know if there exists a polynomial $ P(z, x_1,x_2,\ldots,x_n)$ over the rationals such that the set $$ Z_P = \{z | \exists x_1,\ldots,x_n. P(z, x_1,x_2,\ldots,x_n) = 0 \} \subsetneq \mathbb Q ...
afiori's user avatar
  • 163
0 votes
0 answers
195 views

Is the positive existential theory undecidable?

Could you tell if the positive existential theory of $\mathbb{C}[e^{\mu x} \mid \mu \in \mathbb{C}]$ is undecidable in the language $\{+, \cdot , \frac{d}{dx} , 0, 1, e^x\}$ ? How can we prove the (...
Mary Star's user avatar
  • 299
0 votes
0 answers
63 views

Is the question whether a FO formula F has a model of size k (k is a finite number) decidable?

Hi all, can someone please tell me whether the question whether a FO formula F has a model of size k (where k is a finite number) is decidable? Thanks in advance! TL
user29224's user avatar